LSAT and Law School Admissions Forum

Get expert LSAT preparation and law school admissions advice from PowerScore Test Preparation.

User avatar
 Dave Killoran
PowerScore Staff
  • PowerScore Staff
  • Posts: 5852
  • Joined: Mar 25, 2011
|
#85991
Complete Question Explanation
(The complete setup for this game can be found here: lsat/viewtopic.php?f=302&t=8539)

The correct answer choice is (C)

If G is presented third, the possible positions for the sequence are immediately affected. Specifically, F and H cannot be presented after G because then there would not be enough room to present K, L, and J. Hence, F and H must be presented first and second:
Capture21.PNG
Accordingly, answer choice (C) is correct.

Answer choice (E) is incorrect because there would be no viable placement option for the KL block.
You do not have the required permissions to view the files attached to this post.

Get the most out of your LSAT Prep Plus subscription.

Analyze and track your performance with our Testing and Analytics Package.